¿Qué suposiciones sobre la acción hacemos o abandonamos en la transición de la mecánica clásica a la mecánica cuántica a la teoría cuántica de campos?

Estoy leyendo Teoría del campo cuántico para aficionados superdotados y siento que no tengo una buena comprensión de cómo el Lagrangiano y la acción se usan de manera diferente en (1) la mecánica clásica (2) la mecánica cuántica y (3) la teoría del campo cuántico . No son tanto las matemáticas como que no entiendo lo que representan físicamente.

Por ejemplo, en la mecánica clásica, la acción se utiliza (por ejemplo) en relación con la trayectoria de la partícula. Otro ejemplo que estoy familiarizado con la electrodinámica clásica y el uso de la densidad de Lagrangian L para encontrar ecuaciones de movimiento. Para la mecánica cuántica, pensé que no podíamos conocer la trayectoria. Pensé que confiábamos en una teoría del operador por la cual teníamos | ψ ( t ) = tu ( t ) | ψ ( 0 ) . Ahora, en mi libro qft, escucho que hay algo llamado propagador. El primer ejemplo que encontré es un propagador de función de onda.  

ϕ ( X , t X ) = d y GRAMO + ( X , t X , y , t y ) ϕ ( y , t y )
dónde GRAMO + ( X , t X , y , t y ) = θ ( t X t y ) X ( t X ) | y ( t y ) Y usamos esto básicamente para determinar la amplitud en ϕ ( y , t y ) y luego un punto posterior en la evolución del tiempo ϕ ( X , t X ) del sistema. Todavía no he llegado a la noción de propagadores de campo. Aun así, sentí que estaba entendiendo las cosas. Pero me salté un poco y encontré la ecuación
GRAMO ( X , t X , y , t y ) = D [ q ( t ) ] Exp ( i t X t y d t L [ q ( t ) ] )
Se supone que esto se usa para sumar todas las posibles trayectorias de una partícula que viaja entre dos puntos del espacio-tiempo. Pero ahora estoy muy confundido porque pensé que no podíamos conocer las trayectorias en la mecánica cuántica. Y si no podemos conocerlos en QM, ciertamente no deberíamos hacerlo en QFT. Sin embargo, esta integral parece estar diciendo que estamos sumando sobre trayectorias. Para mí eso no tiene sentido si no puedo conocer las trayectorias.

Mi pregunta

¿Alguien puede resumir brevemente cómo la noción de acción y Lagrangiana se usan de manera diferente?

Pero la integral es sobre todas las trayectorias con las condiciones de contorno apropiadas, no solo la que verifica las ecuaciones clásicas de movimiento, y proporciona el propagador cuántico correcto [tautológicamente, si define QM a partir de la integral de trayectoria]. Por lo tanto, no necesita conocer una trayectoria para alimentar la integral de trayectoria, y no produce ninguna.
No estoy seguro de seguir, así que disculpe si lo siguiente no está relacionado: ¿Está esto relacionado con la idea de que la amplitud puede ser distinta de cero fuera del cono de luz frontal? O más bien, ¿implica esa idea un camino no satisfecho por la acción clásica que sí se satisface en la QFT?
@StanShunpike: Eso tiene que ver con el principio de incertidumbre.
@StanShunpike: estoy elaborando una respuesta sustancial. Para mostrar realmente lo que está pasando, derivaré la integral de trayectoria. Además, encontré algunos diagramas. (¡Vaya fotos!)
Bueno, sí, una teoría clásica permanece dentro del cono de luz ya que las ecuaciones clásicas de movimiento imponen la causalidad. Pero dado que la integral de trayectoria tiene contribuciones de trayectorias que no son soluciones a las ecuaciones clásicas de movimiento, puede tener una amplitud que no desaparece fuera del cono de luz. (La teoría cuántica sigue siendo causal: en la teoría cuántica, la causalidad no es lo mismo que el propagador que se desvanece fuera del cono de luz. Más bien, tenemos que mirar los conmutadores de los observables).
En segundo lugar, la frase "un camino no satisfecho por la acción clásica" no tiene sentido. La acción es un funcional que toma un camino en el espacio fase y da un número. El principio de acción estacionaria da ecuaciones de movimiento que determinan la trayectoria física del sistema clásico.

Respuestas (1)

En NRQM, representamos partículas por un paquete de ondas localizado ψ , llamada función de onda. Aproximadamente, decimos que la partícula clásica se encuentra en el "pico" del paquete de ondas. Decimos que no podemos conocer el camino porque la función de onda puede ser distinta de cero en varios lugares. En lugar de poder decir de manera determinista dónde está la partícula en la mecánica clásica, solo podemos dar una distribución de probabilidad | ψ | 2 .

Ahora mostraré, por medio de una historia apócrifa de Feynman, que las integrales de trayectoria son bastante naturales. (Leí esta historia en QFT Nut de A. Zee).

ingrese la descripción de la imagen aquí

Hace mucho tiempo, en una clase de QM, el profesor habló sobre el experimento de la doble rendija. Una partícula emitida por S en el momento t = 0 pasa por uno de los dos agujeros A 1 , A 2 y se detecta en O en el momento t = T . La amplitud viene dada por el principio de superposición, como la suma de las amplitudes S A 1 O y S A 2 O .

De repente, Feynman preguntó: "¿Qué pasa si perforamos un tercer agujero?" El profesor respondió: "La amplitud de todo el proceso es ahora la suma de las tres amplitudes separadas". Feynman, siendo Feynman, le pregunta al profesor qué sucede si perforamos más agujeros en la pantalla. Obviamente, simplemente sumamos sobre los agujeros . Dejar A denota la amplitud. Entonces

A ( detectado en  O ) = i A ( S A i O )

Pero Feynman insiste: "¿Qué pasa si ahora agregamos una segunda pantalla con algunos agujeros?"Figura 2

El profesor dice algo como: "Se toma la amplitud para ir de S a A i en la primera pantalla, luego a B j en la segunda pantalla, luego a O y luego suma i y j ."

Feynman no ha terminado: "¿Qué pasa si pongo una tercera o cuarta pantalla? ¿Qué pasa si pongo una pantalla y taladro una cantidad infinita de agujeros para que la pantalla ya no esté allí?" (Bastante zen.)

Lo que mostró Feynman es que aunque sólo hubiera un espacio vacío entre la fuente y el detector, la amplitud para que la partícula pasara por cada uno de los agujeros en cada una de las (inexistentes) pantallas. En otras palabras, tenemos que sumar la amplitud para que la partícula se propague desde la fuente hasta el detector siguiendo todos los caminos posibles entre la fuente y el detector.

ingrese la descripción de la imagen aquí

A ( partícula para ir de  S  a  O  a tiempo  T ) = caminos A ( partícula para ir de  S  a  O  a tiempo  T  siguiendo un camino particular )

Ahora hacemos estas ideas matemáticamente precisas. En QM, la amplitud para propagarse desde un punto q I a un punto q F a tiempo T se rige por el operador unitario mi i H T , dónde H es el hamiltoniano. Más precisamente, la amplitud es

A = q F | mi i H T | q I q F | tu ( T ) | q I
Desde tu ( T ) es exponencial, podemos escribirlo como el producto
tu ( T ) = i = 1 norte tu ( d t ) = i = 1 norte mi i H d t ,
dónde d t = T / norte . Sustituye esto en la amplitud
A = q F | i = 1 norte tu ( d t ) | q I
Ahora inserta norte 1 Copias de
I = d q | q q |
entre cada factor de tu ( d t ) :
A = j = 0 norte 1 i = 1 norte 1 d q i q j + 1 | tu ( d t ) | q j
q j = q ( t j ) q I = q 0 q F = q norte
Concentrarse en q j + 1 | tu ( d t ) | q j . Trabajar con una partícula en un potencial no especificado,
H = PAG 2 2 metro + V ( q )
PAG es el operador de cantidad de movimiento, produce valores propios
PAG | pag = pag | pag
q es el operador de posición, produce valores propios
q | q = q | q
Conecte esto y un operador de identidad,
I = d pag | pag pag |
en el bra-ket:
q j + 1 | tu ( d t ) | q j = q j + 1 | mi i d t ( PAG 2 / 2 metro + V ( q ) ) | q j = mi i d t V ( q j ) d pag q j + 1 | mi i d t ( PAG 2 / 2 metro ) | pag pag | q j
Recordar
q | pag = mi i pag q 2 π
podemos simplificar la integral
mi i d t V ( q j ) d pag mi i d t ( pag 2 / 2 metro ) q j + 1 | pag pag | q j = mi i d t V ( q j ) d pag 2 π mi i d t ( pag 2 / 2 metro ) mi i pag ( q j + 1 q j )
La integral se evalúa como
mi i d t V ( q j ) d pag 2 π mi i d t ( pag 2 / 2 metro ) + i pag ( q j + 1 q j ) = i metro 2 π d t mi [ i metro ( q j + 1 q j ) 2 ] / 2 d t i d t V ( q j ) = C mi i d t { ( metro / 2 ) [ ( q j + 1 q j ) / d t ] 2 V ( q j ) }
Reemplazando esto en nuestra fórmula para los rendimientos integrales de trayectoria
q F | mi i H t | q I = C norte ( i = 1 norte 1 d q i ) Exp { i d t j = 0 norte 1 [ metro 2 ( q j + 1 q j d t ) 2 V ( q j ) ] }
Ahora vamos al límite continuo,
norte d t 0
Entonces
límite d t 0 [ q j + 1 q j d t ] 2 = q ˙ 2
y
límite norte j = 0 norte 1 d t = 0 T d t
Defina también la integral sobre todos los caminos.
D q ( t ) = límite norte C norte i = 1 norte 1 d q i
Obtenemos así la representación integral de trayectoria
q F | mi i H t | q I = D q ( t ) mi i 0 T d t ( 1 2 metro q ˙ 2 V ( q ) )
La integral exponenciada es solo la acción, por lo que podemos escribir la amplitud como
q F | mi i H T | q I = D q mi i S [ q ] = D q Exp ( i 0 T d t L ( q , q ˙ ) )

Ahora podemos derivar el principio de mínima acción. Esta es solo una integral elegante de un exponencial complejo oscilante. Cuando S es estacionario, las fases son similares y se suman constructivamente . Cuando nos alejamos de este equilibrio, las fases varían rápidamente y se suman destructivamente . Así que esperamos la mayor contribución a A venir de caminos por los cuales

d S = 0
Tenga en cuenta que las rutas no clásicas se suman a A , pero la contribución dominante, y por lo tanto el camino promedio, es clásica.

Para que esta sea una teoría cuántica válida, debe obedecer la ecuación de Schroedinger. Ahora demostramos que sí.

Podemos escribir la derivada temporal de un vector de estado en el tiempo t = 0 como

d d t | ψ ( 0 ) = | ψ ( d t ) | ψ ( 0 ) d t
Entonces, dada la ecuación de Schroedinger,
i d d t | ψ ( 0 ) = H | ψ ( 0 ) ,
podemos aproximarnos como
| ψ ( d t ) | ψ ( 0 ) = i d t H | ψ ( 0 )
En el X base, tenemos
ψ ( X , d t ) ψ ( X , 0 ) = i d t [ 1 2 metro 2 2 X + V ( X , 0 ) ] ψ ( X , 0 )
Compare esto con la representación de la integral de trayectoria del mismo orden en d t :
ψ ( X , d t ) = tu ( X , d t ; X , 0 ) ψ ( X , 0 ) d X
tu ( X , d t ; X , 0 ) = X | tu ( d t ) | X
De la derivación de la integral de trayectoria, tenemos
tu ( X , d t ; X , 0 ) = metro 2 π i d t Exp { i [ metro ( X X ) 2 2 d t d t V ( X + X 2 , 0 ) ] }
Entonces
ψ ( X , d t ) = metro 2 π i d t Exp [ i metro ( X X ) 2 2 d t ] Exp [ i d t V ( X + X 2 , 0 ) ] ψ ( X , 0 ) d X
El primer término exponencial oscila rápidamente a excepción del punto estacionario X = X , donde la fase tiene el valor mínimo de cero. Decimos que la región de coherencia en la integral de trayectoria es d S π . Así que la región de coherencia para la primera exponencial, en términos de η = X X ,
metro η 2 2 d t π
o
| η | 2 π d t metro
Así que considera ahora
ψ ( X , d t ) = metro 2 π i d t Exp [ i metro η 2 2 d t ] Exp [ i d t V ( X + η 2 , 0 ) ] ψ ( X + η , 0 ) d η
Trabajamos a primer pedido en d t y de segundo orden en η . ampliamos
ψ ( X + η , 0 ) = ψ ( X , 0 ) + η ψ + 1 2 η 2 ψ +
Exp [ i d t V ( X + η 2 , 0 ) ] = 1 i d t V ( X + η 2 , 0 ) + = 1 i d t V ( X , 0 ) +
(condiciones de pedido η d t deben ser descuidados). Ahora nuestra integral se convierte en
ψ ( X , d t ) = metro 2 π i d t Exp [ i metro η 2 2 d t ] [ ψ ( X , 0 ) i d t V ( X , 0 ) ψ ( X , 0 ) + η ψ + 1 2 η 2 ψ ] d η
Haciendo las integrales, obtenemos
ψ ( X , d t ) = metro 2 π i d t [ ψ ( X , 0 ) 2 π i d t metro d t 2 i metro 2 π i d t metro ψ i d t 2 π i d t metro V ( X , 0 ) ψ ( X , 0 ) ]
o
ψ ( X , d t ) ψ ( X , 0 ) = i d t [ 1 2 metro 2 X 2 + V ( X , 0 ) ] ψ ( X , 0 )
que es simplemente la ecuación de Schroedinger.

La mecánica cuántica integral de caminos concuerda perfectamente con la mecánica ondulatoria.

(Esta publicación es realmente larga. El retraso de PSE es insoportable. No dude en hacer preguntas, pero no escribiré más en este cuadro de respuesta).

Rxn1: Leí A Zee antes y escuché esa historia, pero a través de SE aprendí cómo funciona el operador unitario y ahora que lo mencionas, hace clic ahora. veo lo que d t es finalmente! He estado tratando de averiguar de qué está hablando Zee durante un año. ¡Eso es tan cool! Rxn2: wow, acabo de obtener la parte integral de la ruta. Eso es genial. No me di cuenta de que estaba tan conectado con el asunto del freno del libro de Zee, pero ahora que lo diste, puedo seguir el hilo del pensamiento. Rxn3: Acabo de terminar el post y no puedo creer que haya seguido todo eso. Muchas gracias. Bien explicado y claro.
Espera, ¿estás diciendo que RQM no involucra paquetes de ondas? Reconozco que en la teoría cuántica de campos la función de onda no funciona, pero pensé que los paquetes de ondas funcionaban para RQM
@StanShunpike: RQM también usa paquetes de ondas. Sin embargo, en RQM solemos hablar de campos . Los paquetes de ondas son excitaciones de los campos.
@StanShunpike: Soy consciente de que no abordé QFT en mi publicación. Hacerlo habría sido demasiado largo y habría bloqueado Chrome. (Mathjax representa TEX en tiempo real). Me temo que tendrá que hacer una pregunta aparte si quiere que hable sobre integrales de trayectoria de campo.
@StanShunpike: También se debe tener en cuenta que hay quienes no creen que las integrales de ruta representen algo más que una herramienta formal utilizada para la teoría de perturbaciones en QFT y no se deben tomar en serio conceptualmente.